Merge remote branch 'public/master'
[course.git] / latex / problems / Serway_and_Jewett_4 / problem21.38.tex
1 \begin{problem*}{21.38}
2 The following equations describe an electric circuit:
3 \begin{align}
4  -(220\Omega)I_1 + 5.80\U{V} - (370\Omega)I_2 &= 0 \label{eq.2_L1}\\
5  (370\Omega)I_2 + (150\Omega)I_3 - 3.10\U{V} &= 0 \label{eq.2_L2}\\
6  I_1 + I_3 - I_2 &= 0 \label{eq.2_J}
7 \end{align}
8 \Part{a} Draw a diagram of the circuit.
9 \Part{b} Calculate the unknowns and identify the physical meaning of
10 each unknown.
11 \end{problem*} % problem 21.38
12
13 \begin{solution}
14 \Part{a}
15 Looking at the three equations, we see that the only unknowns are
16 $I_1$, $I_2$, and $I_3$.  That looks like a circuit with current in
17 three branches.
18 \begin{center}
19 \begin{empfile}[2a]
20 \begin{emp}(0,0)
21   numeric dx, dy;
22   dx := .5cm;
23   dy := .5cm;
24   % draw dashed branches (with single CCW spiral)
25   draw (0,0)--(0,dy)--(-dx,dy)--(-dx,0)--(dx,0)--(dx,dy)--(0,dy) dashed evenly;
26   % draw the nodes
27   draw (0,0) withpen pencircle scaled 3pt;
28   draw (0,dy) withpen pencircle scaled 3pt;
29 \end{emp}
30 \end{empfile}
31 \end{center}
32 By looking at Eqn.~\ref{eq.2_J} and identifying it with Kirchhoff's
33 junction rule on junction $A$, we can get current directions.
34 \begin{center}
35 \begin{empfile}[2b]
36 \begin{emp}(0,0)
37   numeric dx, dy;
38   dx := 1cm;
39   dy := 1cm;
40   % draw the nodes
41   draw (0,dy) withpen pencircle scaled 3pt;
42   draw (0,0) withpen pencircle scaled 3pt;
43   puttext.bot("$A$", (0,0));
44   % draw dashed branches (with single CCW spiral)
45   draw (0,0)--(0,dy)--(-dx,dy)--(-dx,0)--(dx,0)--(dx,dy)--(0,dy) dashed evenly;
46   centreof.i((-dx,0), (0,0), cur);
47   current.A(c.i, phi.i, "", "$I_1$");
48   centreof.I((0,0), (0,dy), cur);
49   current.B(c.I, phi.I, "I_2", "");
50   centreof.j((dx,0), (0,0), cur);
51   current.C(c.j, phi.j, "I_3", "");
52 \end{emp}
53 \end{empfile}
54 \end{center}
55 Eqn.~\ref{eq.2_L1} looks like a Kirchhoff's loop rule involving only
56 branches 1 and 2.  The first term $-(220\Omega)I_1$ looks like a
57 $V=IR$ resistor drop in the direction of the current on branch 1, so
58 let's add a $220\Omega$ resistor to branch 1.  Because the voltage
59 drops in our loop equation, we must be moving in the direction of the
60 current.  Continuing through the Eqn.~\ref{eq.2_L1}, we see a constant
61 voltage increase, which looks like we crossed a battery from the
62 negative to positive side, so we'll add that onto branch 1 too.
63 Finally, there is a $-(370\Omega)I_2$ drop which looks like crossing a
64 resistor in the direction of the current on branch 2, so let's add a
65 $370\Omega$ resistor to branch 2.
66 \begin{center}
67 \begin{empfile}[2c]
68 \begin{emp}(0,0)
69   pair N[];
70   numeric dx;
71   dx := 2cm;
72   % draw the left branch components
73   resistor.A(origin, normal, -90, "", "220\ohm");
74   battery.A(R.A.r, -90, "", "5.80 V");
75   N[0] := B.A.p;
76   N[1] := R.A.l;
77   N[2] := N[0] + (dx,0);
78   centreof.i(N[0], N[2], cur);
79   current.A(c.i, phi.i, "", "$I_1$");  
80   % draw the middle branch components
81   N[3] := (xpart N[2], ypart N[1]);
82   centerto.B(R.A.r, R.A.l, dx, res);
83   resistor.B(B, normal, 90, "", "370\ohm");
84   centreof.I(N[2], R.B.l, cur);
85   current.B(c.I, phi.I, "I_2", "");
86   % draw the right branch components
87   N[4] := N[2]+(dx,0);
88   N[5] := (xpart N[4], ypart N[3]);
89   centreof.j(N[4], N[2], cur);
90   current.C(c.j, phi.j, "I_3", "");
91   % draw the wires
92   wire(N[0], N[2], rlsq);
93   wire(N[2], R.B.l, rlsq);
94   wire(N[1], R.B.r, nsq);
95   draw N[2]--N[4]--N[5]--N[3] dashed evenly;
96   % draw the nodes
97   draw N[2] withpen pencircle scaled 3pt;
98   draw N[3] withpen pencircle scaled 3pt;
99   % draw the loop direction
100   imesh((N[0]+N[3])/2, ypart (N[1]-N[0])/4, dx/4, ccw, 90, "");
101 \end{emp}
102 \end{empfile}
103 \end{center}
104
105 Eqn.~\ref{eq.2_L2} looks like another Kirchhoff's loop rule, this time
106 involving only branches 2 and 3.  The first term $-(370\Omega)I_2$
107 looks like a resistor gain \emph{against} the direction of the current
108 on branch 2.  We already have a $370\Omega$ resistor to branch 2, so
109 this term just tells us we're moving upstream against $I_2$.
110 Continuing through the Eqn.~\ref{eq.2_L2}, we see another voltage
111 \emph{gain} $(150\Omega)I_3$.  If we're moving upstream on $I_2$,
112 we'll also be moving upstream on $I_3$, so this voltage gain must be a
113 $150\Omega$ resistor on branch 3.  The last term is a constant votage
114 \emph{drop}, which looks like we crossed a battery from the positive
115 to negative side, so we'll add that onto branch 3 too.
116 \begin{center}
117 \begin{empfile}[2d]
118 \begin{emp}(0,0)
119   pair N[];
120   numeric dx;
121   dx := 2cm;
122   % draw the left branch components
123   resistor.A(origin, normal, -90, "", "220\ohm");
124   battery.A(R.A.r, -90, "", "5.80 V");
125   N[0] := B.A.p;
126   N[1] := R.A.l;
127   N[2] := N[0] + (dx,0);
128   centreof.i(N[0], N[2], cur);
129   current.A(c.i, phi.i, "", "$I_1$");  
130   % draw the middle branch components
131   N[3] := (xpart N[2], ypart N[1]);
132   centerto.B(R.A.r, R.A.l, dx, res);
133   resistor.B(B, normal, 90, "", "370\ohm");
134   centreof.I(N[2], R.B.l, cur);
135   current.B(c.I, phi.I, "I_2", "");
136   % draw the right branch components
137   N[4] := N[2]+(dx,0);
138   N[5] := (xpart N[4], ypart N[3]);
139   centreof.j(N[4], N[2], cur);
140   current.C(c.j, phi.j, "I_3", "");
141   resistor.C(N[4], normal, 90, "", "150\ohm");
142   battery.C(N[5], -90, "\mbox{3.10 V}", ""); 
143   % draw the wires
144   wire(N[0], N[2], rlsq);
145   wire(N[2], R.B.l, rlsq);
146   wire(N[1], R.B.r, nsq);
147   wire(N[2], N[4], nsq);
148   wire(N[3], N[5], nsq);
149   % draw the nodes
150   draw N[2] withpen pencircle scaled 3pt;
151   draw N[3] withpen pencircle scaled 3pt;
152   % draw the loop direction
153   imesh((N[2]+N[5])/2, ypart (N[3]-N[2])/4, dx/4, ccw, 90, "");
154 \end{emp}
155 \end{empfile}
156 \end{center}
157
158 \Part{b}
159 Solve using your method of choice.  With linear algebra:
160 \begin{equation}
161  \begin{pmatrix}
162   5.80\U{V} \\
163   3.10\U{V} \\
164   0
165  \end{pmatrix}
166   =
167  \begin{pmatrix}
168   220\Omega & 370\Omega & 0 \\
169   0 & 370\Omega & 150\Omega \\
170   1 & -1 & 1
171  \end{pmatrix}
172  \begin{pmatrix}
173   I_1 \\
174   I_2 \\
175   I_3
176  \end{pmatrix}
177 \end{equation}
178
179 Inverting the 3x3 matrix,
180  we get
181 \begin{equation}
182  \begin{pmatrix}
183   I_1 \\
184   I_2 \\
185   I_3
186  \end{pmatrix}
187   =
188  \begin{pmatrix}
189   0.0031  & -0.0022 & 0.3267 \\
190   0.0009 & 0.0013 & -0.1942 \\
191   -0.0022 & 0.0035 & 0.4791
192  \end{pmatrix}^{-1}
193  \begin{pmatrix}
194   5.80\U{V} \\
195   3.10\U{V} \\
196   0
197  \end{pmatrix}
198   =
199  \ans{
200   \begin{pmatrix}
201     11.0 \\
202     9.13 \\
203     -1.87
204   \end{pmatrix}
205   \U{mA}
206  }
207 \end{equation}
208
209 With regular algebra, we can save ourselves a bit of work by noticing
210 that this problem is the same as the one we just did (35)!  Well, now
211 we have a battery on the first branch and none on the second, and the
212 batteries are facing down\ldots If we flip the picture over and swap
213 the first and second branches\ldots
214 \begin{center}
215 \begin{empfile}[2e]
216 \begin{emp}(0,0)
217   pair N[];
218   numeric dx;
219   dx := 2cm;
220   % draw the left branch components (now middle)
221   resistor.A(origin, normal, 90, "", "220\ohm");
222   battery.A(R.A.r, 90, "", "5.80 V");
223   N[0] := B.A.p;
224   N[1] := R.A.l;
225   N[2] := N[0] - (dx,0);
226   centreof.i(N[1], N[0], cur);
227   current.A(c.i, phi.i, "", "$I_1$");  
228   % draw the middle branch components (now left)
229   N[3] := (xpart N[2], ypart N[1]); % (now bottom)
230   centerto.B(R.A.r, R.A.l, -dx, res);
231   resistor.B(B, normal, 90, "", "370\ohm");
232   centreof.I(N[2], R.B.r, cur);
233   current.B(c.I, phi.I, "I_2", "");
234   % draw the right branch components
235   N[4] := N[0]+(dx,0);
236   N[5] := (xpart N[4], ypart N[3]); % (now bottom)
237   centreof.j(N[5], N[4], cur);
238   current.C(c.j, phi.j, "I_3", "");
239   resistor.C(N[4], normal, -90, "\mbox{150\ohm}", "");
240   battery.C(N[5], 90, "", "3.10 V"); 
241   % draw the wires
242   wire(N[0], N[2], rlsq);
243   wire(N[1], R.B.l, rlsq);
244   wire(N[2], R.B.r, nsq);
245   wire(N[2], N[4], nsq);
246   wire(N[3], N[5], nsq);
247   % draw the nodes
248   draw N[0] withpen pencircle scaled 3pt;
249   draw N[1] withpen pencircle scaled 3pt;
250 \end{emp}
251 \end{empfile}
252 \end{center}
253 Alright, now it looks like the figure in Problem 35, except that the
254 things labeled $X_1$ and $X_2$ are reversed.  We can take our analytic
255 solution to 35 (see the linear algebra notes) and exchange $1
256 \leftrightarrow 2$ giving
257
258 \begin{align}
259    \frac{\frac{\epsilon_3}{R_3}+\frac{\epsilon_1}{R_1}}{\frac{R_2}{R_3}+\frac{R_2}{R_1}+1} &= I_2 = \ans{9.13\U{mA}} \\
260    \frac{\epsilon_1}{R_1} - \frac{1}{R_1}\frac{\frac{\epsilon_3}{R_3}+\frac{\epsilon_1}{R_1}}{\frac{1}{R_3}+\frac{1}{R_1}+\frac{1}{R_2}} &= I_1 = \ans{11.0\U{mA}} \\
261    \frac{\epsilon_3}{R_3} - \frac{1}{R_3}\frac{\frac{\epsilon_3}{R_3}+\frac{\epsilon_1}{R_1}}{\frac{1}{R_3}+\frac{1}{R_1}+\frac{1}{R_2}} &= I_3 = \ans{-1.87\U{mA}}
262 \end{align}
263 \end{solution}